divide the circumference of a pumpkin by its diameter and what do you get?

Answers

Answer 1

Dividing the circumference of a pumpkin by its diameter gives a value approximately equal to pi (π)

When you divide the circumference of a pumpkin by its diameter, you get a value that is approximately equal to the mathematical constant pi (π), which is approximately 3.14159.

This is because pi represents the ratio of the circumference of a circle to its diameter, and a pumpkin is roughly spherical in shape. So, no matter how big or small the pumpkin is, if you measure its circumference and diameter and divide them, the result will be very close to pi.

Mathematically, this can be represented by the formula

pi ≈ circumference / diameter

Learn more about Pi here

brainly.com/question/16277677

#SPJ4


Related Questions

the radius of a circle is changing at .5 cm/sec. find the rate of change of the area when the radius is 4 cm.

Answers

The rate of change of the area of a circle when the radius is 4 cm is 4π cm2/sec.

This can be calculated using the formula for the area of a circle (A = πr2) and the chain rule for derivatives. The chain rule states that when the radius (r) changes, the area of a circle (A) is equal to 2πr times the rate of change of the radius (dr/dt).

Therefore, the rate of change of the area of a circle when the radius is 4 cm is equal to 2π(4 cm) × (0.5 cm/sec) = 4π cm2/sec.

Note that if the rate of change of the radius were different, the rate of change of the area would also be different. This formula can be used to calculate the rate of change of the area of a circle at any given radius, as long as the rate of change of the radius is known.

For more such questions on Rate of change of area.

https://brainly.com/question/30435679#

#SPJ11

solve for x (Please leave an explanation)

Answers

The value of x for the angle 100 + x under the top parallel line is equal to -10.

What are angles formed by a pair of parallel lines cut by a transversal line?

When a transversal line intersects a pair of parallel lines, several angles are formed which includes: Corresponding angles, vertical angles, and alternate angles.

The angle 100 + x formed by the top parallel line and the transversal line perpendicular to to it is equal to 90° so we can solve for the value of x as follows:

100 + x = 90

subtract 100 from both sides

x = 90 - 100

x = -10.

Therefore, the value of x for the angle 100 + x under the top parallel line is equal to -10.

Know more about angles here:https://brainly.com/question/24607467

#SPJ1

Noah was at home. He got on his bike and rode to his friends

Answers

Answer:

what's your exact question

Answer:

can u pls type the full question

Proportions

Two plus x divided by twelve equals one dived by three. Solve for x.

Answers

Two plus x divided by twelve equals one divided by three

Case 1 :

Rewrite into numbers : 2 + x /12 = 1/3

-> x/12 = 1/3 - 2 = -5/3

-> x = -5/3 x 12 = -20

Case 2 :

Rewrite into numbers : (2 + x)/12 = 1/3

-> 2 + x = 1/3 x 12 = 4

-> x = 4 - 2 = 2

i dont know if you meant it the right way or the wrong way but ill just put them both

x=2

Step-by-step explanation:

(2+x)/12=1/3

3(2+x)=12

2+x=4

x=4-2

x=2

The point on the graph represents Ann's location. She is using a metal detector on the beach to see what she can find. Each unit on the graph represents 2 feet. A pile of bottle caps is located at (4, -10). Find the length of the most direct path between Ann and the pile of bottle caps. Round to the nearest whole number.

Answers

Answer:

30 feet

Step-by-step explanation:

Coordinates of Ann: (-4,3)

Coordinates of bottle caps: (4,-10)

Distance from Ann to bottle caps can be found out using the distance formula:
[tex]x_2=4, x_1=-4\\y_2=-3,y_1=-10\\Distance=\sqrt{(x_{2}-x_{1})^2 + (y_{2}-y_{1})^2 } \\=\sqrt{(4-(-4))^2 + ((-10)-3)^2} \\=15.26\\15\text{ is the answer}[/tex]

A right triangle is shown. What is the approximate length of the hypotenuse of the triangle? ​

Answers

Answer:

Option D.

Step-by-step explanation:

To find hypotenuse [tex]c[/tex] use formula:

                                    [tex]\text{Cos(B)}=\frac{a}{c}[/tex]

       After substituting [tex]B=62^0[/tex]  and a = 7 we have:

                                 [tex]\text{cos}(62^o)=\frac{7}{c}[/tex]

                                     [tex]0.4695=\frac{7}{c}[/tex]

                                         [tex]c=\frac{7}{0.4695}[/tex]

                                          [tex]c=14.9104[/tex]

suppose there is $600 in the account with an annual interest rate of 4%. after how many years will the amount triple?

Answers

it will take approximately 22.56 years for the amount to triple.

The given information for this problem is that there is an initial investment of $600 in an account with an annual interest rate of 4%. The task is to determine after how many years the amount will triple.Using the compound interest formula, we can find the amount in the account after t years:A = P(1 + r/n)nt Where,A = final amount in the account, P = initial amount in the account r = annual interest rate ,n = number of times the interest is compounded per year ,t = time in years.

From the problem statement, we know that the initial amount, P, is $600 and the annual interest rate, r, is 4%. Let's assume that the interest is compounded annually, i.e., n = 1.Substituting these values in the formula, we get:A = $600(1 + 0.04/1)1t Simplifying this expression,A = $600(1.04)t.

Taking the ratio of the final amount to the initial amount, we get: 3P = $600 × 3 = $1800. Therefore,A/P = 3 = (1.04)t.Dividing both sides by P, we get:3 = (1.04)t ln(3) = ln(1.04)t. Using the logarithmic property, we can bring down the exponent to the front:ln(3) / ln(1.04) = t Using a calculator, we get ≈ 22.56. Therefore, it will take approximately 22.56 years for the amount to triple.

To know more about compound interest, click here:

https://brainly.com/question/22621039

#SPJ11

Mai poured 2. 6 liters of water into a partially filled pitcher. The pitcher then contained 10. 4 liters. How much water did the pitcher contain before Mai added more water?

Answers

The amount of water that the pitcher contained before Mai added more water was 7.8 liters.

Let x be the amount of water the pitcher contained before Mai added more water.

When Mai poured 2.6 liters of water, the total amount of water in the pitcher became x + 2.6 liters.

According to the problem, the pitcher then contained 10.4 liters of water. Therefore, we can write:

x + 2.6 = 10.4

Subtracting 2.6 from both sides, we get:

x = 7.8

Therefore, the pitcher contained 7.8 liters of water before Mai added more water.

To learn more about liters Click here:
brainly.com/question/14780251

#SPJ4

a children's liquid medicine contains 100 mg of the active ingredient in 5 ml . if a child should receive 300 mg of the active ingredient, how many milliliters of the medicine should the child be given? for the purposes of this question, assume that these numbers are exact.

Answers

The child should be given 15 ml of the medicine to receive 300 mg of the active ingredient.

The given problem requires us to determine the number of milliliters of a liquid medicine that a child should receive in order to obtain a specific dosage of the active ingredient. We are given that the medicine contains 100 mg of the active ingredient in 5 ml.

The child needs to receive 300 mg of the active ingredient, and there are 100 mg of the active ingredient in 5 ml of the medicine. Therefore, the child should be given:

[tex]\frac{300 mg}{100mg/5ml} = \frac{300\text{ mg} \times 5\text{ ml}}{100\text{ mg}} = 15\text{ ml}$$[/tex]

So the child should be given 15 ml of the medicine to receive 300 mg of the active ingredient.

Learn more about Liquid Measurements:

https://brainly.com/question/7744911
#SPJ4

Triangle ABC is shown below. Describe how we could use circles to determine whether this is an equilateral triangle.

Answers

In Triangle ABC, we can draw a circle around each vertex of the triangle.

If the radii of all three circles are equal, then Triangle ABC is an equilateral triangle.

What is radii?

Radii is the plural form of radius. It is half of the diameter and the length of the radius is used to calculate the area and circumference of the circle.

To determine whether a triangle is an equilateral triangle, we can use circles.

In this method, we draw three circles, one around each vertex of the triangle.

We then measure the radii of the circles, ensuring that they are all equal. If the radii of the three circles are equal, then the triangle is an equilateral triangle.

In Triangle ABC, we can draw a circle around each vertex of the triangle. Then, we could use a measuring tool to measure the radii of each circle. If the radii of all three circles are equal, then Triangle ABC is an equilateral triangle.

This method of using circles to determine whether a triangle is an equilateral triangle is simple and efficient. It does not require any complex calculations, and it is easy to understand.

For more questions related to equilateral triangle

https://brainly.com/question/17264112

#SPJ1

Help please, Which value of x satisfies the equation 7/3(x+9/28)=20

Answers

Answer:

Step-by-step explanation:

[tex]\frac{7}{3} (x+\frac{9}{28} )=20[/tex]    

[tex]7 (x+\frac{9}{28} )=60[/tex]     (multiplied both sides by 3)

[tex]x+\frac{9}{28} =\frac{60}{7}[/tex]          (divided both sides by 7)

[tex]x=\frac{60}{7}-\frac{9}{28}=\frac{240}{28}-\frac{9}{28}=\frac{231}{28} =8.25[/tex]     (subtracted [tex]\frac{9}{28}[/tex] both sides and solved)

The triangles are similar. Find the value of x.

Answers

Since the triangles are similar, the value of x is equal to: C. 18 units.

What are the properties of similar triangles?

In Mathematics, two (2) triangles are said to be similar when the ratio of their corresponding side lengths are equal and their corresponding angles are congruent.

By applying the properties of similar triangles, we have the following ratio of corresponding side lengths;

AC/RS = AB/RT

By substituting the given side lengths into the above equation, we have the following:

x/24 = 24/32

By cross-multiplying, we have the following;

32x = 24(24)

32x = 576

x = 576/32

x = 18 units.

Read more on triangle here: brainly.com/question/22891315

#SPJ1

Missing information:

The question is incomplete and the complete question is shown in the attached picture.

which of the following conditions must be met to conduct a two-proportion significance test? the populations are independent. the probabilities of success multiplied by the sample sizes are greater than or equal to 10 and the probabilities of failure multiplied by the sample sizes are greater than or equal to 10 for each population. the sample sizes are greater than 30.

Answers

The following conditions must be met to conduct a two-proportion significance test:

the populations are independent, the probabilities of success multiplied by the sample sizes are greater than or equal to 10 and the probabilities of failure multiplied by the sample sizes are greater than or equal to 10 for each population, and the sample sizes are greater than 30.

The two-proportion significance test is a hypothesis test that compares the proportions of two independent populations.

To conduct the two-proportion significance test, the following conditions must be met:

Populations must be independent.

Sample sizes are greater than 30.

The probabilities of success multiplied by the sample sizes are greater than or equal to 10 and the probabilities of failure multiplied by the sample sizes are greater than or equal to 10 for each population.

The sample size should be large enough so that the sampling distribution of the sample proportion is nearly normal. The sample sizes should be large enough so that the central limit theorem can be applied.

In short, to conduct a two-proportion significance test, the populations must be independent, the probabilities of success multiplied by the sample sizes are greater than or equal to 10 and the probabilities of failure multiplied by the sample sizes are greater than or equal to 10 for each population, and the sample sizes are greater than 30.


To know more about two-proportion significance test refer here:

https://brainly.com/question/30255520#

#SPJ11

To make cleaning easier, a rectangular horse trough will be lined with plastic. The trough is 40 inches long, 14 inches wide, and 24 inches deep. How many square inches of plastic are needed to line the trough? Count only the trough's five faces. A net containing 5 rectangles. Two rectangles have length of 40 inches and width of 14 inches. Two rectangles have length of 14 inches and width of 24 inches. One rectangle has length of 40 inches and width of 24 inches.

Answers

Using the area formula for the rectangle, we can find that 2752 in² of plastic is needed to line the trough.

Define area?

To determine the area a rectangle occupies within its perimeter, apply the formula for calculating a rectangle's area. Multiplying the length by the width yields the area of a rectangle (breadth).

As a result, the area of a rectangle with the length and breadth l and w, respectively, is calculated as follows. L × W = the rectangle's area. Hence, the area of a rectangle is equal to (length width).

Now in the given question,

We have 5 faces of the cuboid.

Now to find the total area of the required space we have to find the area of all the rectangles.

Area of rectangle with dimensions, l = 40inches and b = 14 inches.

Area = l × b

= 40 × 14

= 560in²

Now there are 2 rectangles with the same dimensions, so the total area = 560 + 560 = 1120in².

Now area of rectangles with dimensions, l = 14 inches and b = 24 inches.

Area = l × b

= 14 × 24

= 336in².

There are 2 rectangles with the same dimensions, so area = 336 + 336 = 672in².

Area of the final rectangle = l × b

= 40 × 24

= 960in².

So, the total required area = 1120 + 672 + 960 = 2752in².

To know more about area of rectangles, visit:

https://brainly.com/question/16309520

#SPJ1

How to turn 0. 1212121212 into a simplified fraction

Answers

Answer:

  4/33

Step-by-step explanation:

You want to write 0.1212...(repeating) as a simplified fraction.

Repeating decimal

A repeating decimal beginning at the decimal point can be made into a fraction by expressing the repeating digits over an equal number of 9s.

Here, there are 2 repeating digits, so the basic fraction is ...

  12/99

This can be reduced by removing a factor of 3 from numerator and denominator:

  [tex]0.\overline{12}=\dfrac{12}{99}=\boxed{\dfrac{4}{33}}[/tex]

__

Additional comment

Formally, you can multiply any repeating decimal by 10 to the power of the number of repeating digits, then subtract the original number. This gives the numerator of the fraction. The denominator is that power of 10 less 1.

  0.1212... = (12.1212... - 0.1212...)/(10^2 -1) = 12/99

Doing this multiplication and subtraction also works for numbers where the repeating digits don't start at the decimal point. Finding a common factor with 99...9 may not be easy.

You can also approach this by writing the number as a continued fraction. The basic form is ...

  [tex]x=a+\cfrac{1}{b+\cfrac{1}{c+\cdots}}[/tex]

where 'a' is the integer part of the original number, and b, c, and so on are the integer parts of the inverse of the remaining fractional part. The attachment shows how this works for the fraction in the problem statement.

A calculator cannot actually represent a repeating decimal exactly, so error creeps in and may eventually become significant.

if the odds on a bet are 16:1 against, what is the probability of winning? express your answer as a fraction.

Answers

The probability of winning is 1/17, which can also be expressed as a decimal (approximately 0.059) or as a percentage (approximately 5.9%).

The odds on a bet represent the ratio of the probability of winning to the probability of losing. In this case, the odds are 16:1 against winning, which means that the probability of winning is 1 out of 16.

To express this probability as a fraction, we can use the formula:

Probability of winning = 1 / (odds + 1)

Plugging in the given odds, we get:

Probability of winning = 1 / (16 + 1)

Probability of winning = 1/17

In this case, the odds of 16:1 against winning correspond to a probability of 1/17, which represents the chance of winning the bet.

To learn more about probability click on,

https://brainly.com/question/29111123

#SPJ4

Jason has a block of clay that is made up of two rectangular pieces of
different colors. Find the volume of the block of clay.
The measurements of the two clay blocks are:
6 cm
4 cm
3 cm
5 cm

Answers

Answer: 132 cubic centimeters

Step-by-step explanation:

To find the volume of the block of clay, we need to add the volumes of the two rectangular pieces.

The volume of a rectangular solid can be found by multiplying its length, width, and height. Let's call the first rectangular piece A and the second rectangular piece B. Then the dimensions of A are 6 cm (length), 4 cm (width), and 3 cm (height), and the dimensions of B are 5 cm (length), 4 cm (width), and 3 cm (height).

The volume of A is:

Volume of A = length x width x height = 6 cm x 4 cm x 3 cm = 72 cubic centimeters

The volume of B is:

Volume of B = length x width x height = 5 cm x 4 cm x 3 cm = 60 cubic centimeters

So the total volume of the block of clay is:

Volume of block = Volume of A + Volume of B = 72 cubic centimeters + 60 cubic centimeters = 132 cubic centimeters

Therefore, the volume of the block of clay is 132 cubic centimeters.

a box contains 8 red balls and 8 blue balls, and 4 balls are taken at random without replacement. what is the probability that 2 red balls and 2 blue balls are taken?

Answers

The probability that 2 red balls and 2 blue balls are taken from the box is 3/7. This can be expressed mathematically as [tex](8C2 * 8C2) / (16C4) = 3/7[/tex].

To better understand this probability, let's look at an example. Say there are 8 red balls and 8 blue balls in the box. This can be represented as:

R = 8, B = 8

We want to determine the probability of taking 2 red balls and 2 blue balls out of the box. To do this, we need to calculate the total number of ways of selecting 4 balls from the box (16 balls in total) and then calculate the total number of ways of selecting 2 red and 2 blue balls out of the box.

The total number of ways of selecting 4 balls from the box can be expressed as (16C4). This is calculated by dividing the number of ways of selecting 4 balls out of 16 (16!) by the number of ways of arranging those 4 balls in any order (4!):

[tex](16C4) = 16! / 4! = 1820[/tex]

The total number of ways of selecting 2 red and 2 blue balls out of the box can be expressed as (8C2 * 8C2). This is calculated by multiplying the number of ways of selecting 2 red balls out of 8 (8C2) by the number of ways of selecting 2 blue balls out of 8 (8C2):

[tex](8C2 * 8C2) = 8C2 * 8C2 = 28[/tex]

The probability of taking 2 red balls and 2 blue balls out of the box is then the ratio of the number of ways of selecting 2 red and 2 blue balls out of the box (28) to the total number of ways of selecting 4 balls from the box (1820):

P(2 red balls, 2 blue balls) = 28 / 1820 = 3/7

In conclusion, the probability of taking 2 red balls and 2 blue balls out of a box containing 8 red balls and 8 blue balls is 3/7.

See more abut probability at: https://brainly.com/question/24756209

#SPJ11

44.0183 rounded to the nearest thousands

Answers

44.0180 since 8 cannot be rounded because of 3.

which value of n makes the equation true
-[tex]\frac{1}{2}n=-8[/tex]

Answers

Answer:

16

Step-by-step explanation:

When we divide the entire equation by 1/-2 to get rid of the coefficient of n we get n = 16.

8x 2 + [ 3x3-8] = with explanation pls and its due in six minutes

Answers

Answer: 16x+3x^3−8

Please mark me brainliest :)

If g(x) = 1 – 2x + 3x2, find the average rate of change of the function as x varies from 2 to 5

Answers

The average rate of change of the function as x varies from 2 to 5 is 22.

Given function: g(x) = 1 – 2x + [tex]3x^2[/tex]

To find the average rate of change of the function as x varies from 2 to 5.

Solution: We are given a function: g(x) = 1 – 2x + [tex]3x^2[/tex]

The average rate of change of the function as x varies from a to b is given by:

Average rate of change = f(b) - f(a) / b - a

Let a = 2 and b = 5

We have to find the average rate of change of g(x) as x varies from 2 to 5.

So, the average rate of change of g(x) is given by:

Average rate of change = g(5) - g(2) / 5 - 2

= [1 - 2(5) + 3([tex]5^2[/tex])] - [1 - 2(2) + 3([tex]2^2[/tex])] / 3

= [1 - 10 + 75] - [1 - 4 + 12] / 3= 66 / 3= 22

Therefore, the average rate of change of the function as x varies from 2 to 5 is 22.

An average rate of change is the amount that the function changes on average over a specified interval.

The formula for average rate of change is given as the change in the function value divided by the change in x value for two distinct points on the function.

For similar question on function.

https://brainly.com/question/22340031

#SPJ11

what is the probability that the first question she gets right is question number 4? group of answer choices

Answers

The probability that the first question she gets right is question number 4, is 0.1054.

Number of options there are for a single query = 4

P(guessing correct answer for a single question) = 1/4

P(guessing correct answer for a single question) = 0.25

Probability of getting correct answer P(correct) = 0.25

Probability of getting wrong answer P(wrong) = 1 - Probability of getting correct answer

Probability of getting wrong answer P(wrong) = 1 - 0.25

Probability of getting wrong answer P(wrong) = 0.75

So, the probability that the first question she gets right is question number 4 = Probability of getting 1st question wrong × Probability of getting 2nd question wrong × Probability of getting 3rd question wrong × Probability of getting 4th question right

The probability that the first question she gets right is question number 4 = 0.75 × 0.75 × 0.75 × 0.25

The probability that the first question she gets right is question number 4 = 0.1055

To learn more about probability link is here

brainly.com/question/31120123

#SPJ4

The complete question is:

In a multiple choice exam, there are 5 questions and 4 choices for each question (a, b, c, d). Nancy has not studied for the exam at all and decides to randomly guess the answers. (Round your answers to four decimal places.)

What is the probability that the first question she gets right is question number 4?

if the matrix product a1b is known, how could you calculate b1a without necessarily knowing what a and b are?

Answers

We can calculate its product by taking the dot product of each row of B1A and each column of A1B. In this way, we can calculate B1A without knowing the values of A and B.

The matrix product of two matrices, A and B, is defined as the matrix C, where C = AB. To calculate the product of two matrices, we must take the dot product of each row of A and each column of B. If we are given a matrix product A1B, then we can calculate B1A without necessarily knowing what A and B are.

To do so, we must first invert the matrix A1B. We can do this by solving a system of equations. We can set up this system of equations by treating the entries of A1B as the coefficients in a system of equations, and solving for the entries of B1A. Once we have found the inverse, we can calculate the matrix B1A.

Finally, once we have the matrix B1A, we can calculate its product by taking the dot product of each row of B1A and each column of A1B. In this way, we can calculate B1A without knowing the values of A and B.

See more about matrix at: https://brainly.com/question/14414517

#SPJ11

The measure of the angle turns through 3/5 of 360°, true or false

Answers

Answer:

false, 216° turns 3/5 in a 360° rotation

9 km
7 km
3 km
3 km
3 km
2 km
8 km
9 km
3 km
7 km

Answers

Answer: what do you mean? I need more info-

Step-by-step explanation:

I can answer it with more info :)

Determine whether segment lengths form a triangle. If so, classify the triangle as acute, right or obtuse.

1. 10, 7, sqrt(658)

Answers

Answer:

it is a triangle bc it has angles of points

Step-by-step explanation:

Please help me ill give
Brainliest

Answers

Answer:

A

Step-by-step explanation:

Since both the x and y coordinates changed signs, this is a reflection over the origin. A reflection over the origin is a reflection across both axes.

Answer: A

Help me with these please!!

Answers

The angle ABD is 35 degrees, AC is 20 units long, and AB is 29 units long.

What in mathematics is an angle?

An angle is created by combining two rays (half-lines) that have a common terminal. The angle's vertex is the latter, while the rays are alternately referred to as the angle's legs and its arms.

Triangle ABD's angle ABC is one of its outside angles, making it equal to the sum of the opposing interior angles.

Angle ABC = Angle ABD + Angle ACD

replacing the specified values:

110° = Angle ABD + 75°

Simplifying:

Angle ABD = 110° - 75°

Angle ABD = 35°

Due of their shared angles, the two triangles are comparable. This fact can be used to establish a ratio between the corresponding sides:

AC / CD = AB / BD

replacing the specified values:

AC / 10 = 16 / 8

Simplifying:

AC = 20

To know more about angle visit:-

brainly.com/question/28451077

#SPJ1

Complete Question:

Find the angle ∠ABD jn the given figure

Align the variables in the equations.
2x - 3y = 9
1-6x +9y = -7

Answers

The variables in the equations are aligned as follows:   2x - 3y = 9 and

2x - 3y = 8/3

What does it mean by align a system of linear equation?

When we talk about aligning a system of linear equations, we mean rearranging the equations so that the variables are in the same order and have the same coefficients. This is done to make it easier to apply methods for solving systems of equations, such as substitution or elimination.

In a system of linear equations, each equation typically involves two or more variables. The variables may have different coefficients in each equation, and they may appear in a different order. Aligning the system involves rearranging the equations in a way that puts the variables in the same order, with the same coefficients.

Align the variables in given the system of linear equations :

To align the variables in the given equations, we need to rearrange the second equation so that the coefficients of  and  are the same as in the first equation.

To align the variables in the equations, we need to rearrange the terms so that the x, y, and constant terms are all grouped together.

Starting with the first equation:

2x - 3y = 9

We can rearrange this as:

2x = 3y + 9

Now we can divide both sides by 2 to get x by itself:

x = (3/2)y + 4.5

Now let's move on to the second equation:

1-6x +9y = -7

We can rearrange this as:

-6x + 9y = -8

Next, we'll divide both sides by -3 to get x by itself:

2x - 3y = 8/3

Now both equations are in the form of:

ax + by = c

where a, b, and c are constants. The aligned equations are:

2x - 3y = 9

2x - 3y = 8/3

To know more about variables visit :

brainly.com/question/29583350

#SPJ1

Other Questions
information flow can be altered by mutation. describe three different types of mutations and their effect on protein synthesis If you were studying the functions of skeletal muscle, you would be studying all of the following except:protecting internal organs.movement.holding the head erect.production of blood cells.helping maintain a constant body temperature. in the number 240.149, how does the value of the 4 in the hundredths place compare to the value of the 4 in the tens place? the shortest side of a triangle with angles 50o, 60o, and 70ohas length of 9 furlongs. what is the approximate length, in furlongs, of the longest side? 16. A savings account was worth $1250 at the end of 2010 and worth $1306 at the end of 2011. The linear modelfor the worth of the account is w = 56t+1250, where t is the number of years since the end of 2010.Find an exponential model, in the form of w= a(b)', for the worth of the savings account. Round b to thenearest thousandth.How much greater is the worth predicted by the exponential model than predicted by the linear model at theend of 2020? Round to the nearest cent. un pome sur la nouriture svpp what is scurvy two symptoms of it trevor spends 45 minutes a day in front of an intense light. it is most likely that he is being treated with despite the deteriorating discipline and living conditions at valley forge in the winter of 1777-1778, american troops benefited from Make an expression with 4 terms where 3 of them are like terms.2. Rewrite the expression by combining like terms. How many terms are there?please help Which historical events have caused American authorities to want to surveil persons?There may be more than one correct answer. Be sure to select all that apply. how many moles of naoh will react with 0.50 mol of h2co3?a.0,25 mol NaOHb.0.50 mol NaOHc.1.0 mol NaOhd.2.0 mol NaOH fill in the blank question 8 fill in the blank: when posting in a discussion forum, you should make sure that any articles discussed are ___ to data analytics. 1 point unique well known popular relevant How does a decrease in the price level affect real wealth and aggregate demand? a customer-oriented firm should focus exclusively on satisfying its internal customers. group of answer choices true false which of the following foods is the best source of calcium? view available hint(s)for part a which of the following foods is the best source of calcium? meat milk fruits grains a bond is worth 100$ and grows in value by 4 percent each year. f(x) = Just need the answers A pharmacist mixes 10 grams of a 15% medicine solution with 25 grams of a 10% medicine solution. Suppose we know that after she adds the x grams of pure medicine the pharmacists mixture is 25% medicine solution. Write an equation Vocabulary and GrammarThe question below refers to the selection "Ode to the West Wind" by Percy Bysshe Shelley.Which sentence best paraphrases this passage from "Ode to the West Wind"?.share/The impulse of thy strength... O uncontrollable!a. Share the dominance of your strength!b. Share the ferocity of your strength!APlease select the best answer from the choices providedB...CDc. Share the passion of your strength!d. Share the driving force of your strength!